12
$\begingroup$

First, I have to admit that I have already asked the same question on MSE several days ago. If I am bending any rules, I apologize for that and moderator can delete or close this question without warning. The problem has received a number of upvotes on MSE but with no suggestions or hints provided from the community.

I need to prove the following:

$$\sum_{n = 1}^{p - 1} n^{p - 1} \equiv (p - 1)! + p \pmod {p^2}$$

...with $p$ being an odd prime number. I'm pretty much sure that the statement is correct (I have tested it by computer for many values of $p$ and did not find an exception).

What puzzles me is the fact that the statement is trivial and obviously true for$\pmod p$. The left-hand side is congruent to $-1 \pmod p$ by Fermat's little theorem, and the right-hand side is also congruent to $-1 \pmod p$ by Wilson's theorem.

However, I am unable to "lift the exponent" and go from$\pmod p$ to$\pmod {p^2}$. Maybe the sum on the left could somehow be transformed using the existence of a primitive root$\pmod {p^2}$.

Thanks and happy holidays!

$\endgroup$
3
  • 2
    $\begingroup$ I have already mentoined that the problem is crossposted. The answer is just a hint. The question comes from a number theory book that does not even mention Beronoulli numbers so I suppose there is a simple(r) solution. The hint is pretty useless to me. $\endgroup$
    – Saša
    Dec 31, 2018 at 10:42
  • 1
    $\begingroup$ Did you try to adapt Lagrange's alternate proof of Wilson's theorem? It uses finite differences of the sequence $n^{p-1} $. fr.m.wikipedia.org/wiki/… $\endgroup$ Dec 31, 2018 at 11:30
  • 1
    $\begingroup$ @FrançoisBrunault Thanks for the hint, I'll check that! $\endgroup$
    – Saša
    Dec 31, 2018 at 11:31

4 Answers 4

26
$\begingroup$

The result can be easily proved without using Bernoulli numbers. If $a$ and $b$ are integers not divisible by an odd prime $p$, then \begin{align}(ab)^{p-1}-1=&b^{p-1}(a^{p-1}-1)+(b^{p-1}-1) \\\equiv& (a^{p-1}-1)+(b^{p-1}-1)\pmod {p^2}.\end{align} Thus \begin{align*}\sum_{n=1}^{p-1}(n^{p-1}-1)\equiv& \prod_{n=1}^{p-1}n^{p-1}-1=((p-1)!+1-1)^{p-1}-1 \\\equiv &(p-1)((p-1)!+1)(-1)^{p-2}\equiv(p-1)!+1\pmod{p^2}\end{align*} and hence the desired congriuence follows.

$\endgroup$
1
  • $\begingroup$ The last term is really $1(\mod p^2)$ and not $p(\mod p^2)$? $\endgroup$
    – pisoir
    Aug 19, 2020 at 18:25
11
$\begingroup$

You may use Faulhaber's formula $$ \sum_{k=1}^p k^{p-1}=\frac{p^p}p+\frac12 p^{p-1}+\sum_{k=2}^{p-1} \frac{B_k}{k!}\,(p-1)^{\underline{k-1}}\,p^{p-k}. $$ All summands except this corresponding to $k=p-1$ are divisible by $p^2$ (they are not integers, but corresponding Bernoulli numbers do not have $p$ in denominators). For $k=p-1$ you get $pB_{p-1}$. For evaluating it modulo $p^2$ we may use the formula for Bernoulli numbers via Worpitzky numbers: $$ B_{p-1}=\sum_{k=0}^{p-1} (-1)^k\frac{k!}{k+1}\left\{\matrix{p\\k+1}\right\}. $$ All summands except corresponding to $k=0$, $k=p-1$ are divisible by $p$ (the corresponding Stirling numbers of the second kind are divisible by $p$ dues to the action of the cyclic shifts group on the partitions of $\{1,2,\dots,p\}$). For $k=0$ and $k=p-1$ we get $pB_{p-1}\equiv (p-1)!+p\pmod {p^2}$ as desired.

$\endgroup$
11
$\begingroup$

It is a theorem of Lerch of 1905. An elementary proof can be found in this article of Sondow : https://arxiv.org/abs/1110.3113

$\endgroup$
4
$\begingroup$

One may also use the fundamental theorem of symmetric polynomials applied to the polynomial $f(x_1,\dots,x_{p-1})=x_1^{p-1}+\dots+x_{p-1}^{p-1}+(p-1)x_1\dots x_{p-1}\in \mathbb{Z}[x_1,\dots,x_{p-1}]$. Suppose $f=g(\sigma_1,\dots,\sigma_{p-1})$, where $\sigma_i$'s are elementary symmetric polynomials, and $g$ is a polynomial with integer coefficients. Let $\zeta$ is the $p$-th root of unity. Note that since $f(\zeta,\zeta^2,\dots,\zeta^{p-1})=0$ and all $\sigma_i$'s except for $\sigma_{p-1}$ vanish at $(\zeta,\zeta^2,\dots,\zeta^{p-1})$, there is no monomial $\sigma_{p-1}$ in $g$. It remains to note that $\sigma_i(1,2,\dots,p-1)$ is divisible by $p$ for any $i<p-1$ and so $f(1,2,\dots,p-1)$ is divisible by $p^2$ which easily implies the result.

$\endgroup$

Your Answer

By clicking “Post Your Answer”, you agree to our terms of service and acknowledge you have read our privacy policy.

Not the answer you're looking for? Browse other questions tagged or ask your own question.